Find all $n$ such that $n/d(n) = p$, a prime, where $d(n)$ is the number of positive divisors of $n$












5












$begingroup$



Let $d(n)$ denote the number of positive divisors of $n$. Find all $n$ such that $n/d(n) = p$, a prime.




I tried this, but only I could get two solutions.
I proceeded like this -



Suppose
$$n = p^r cdot p_1^{r_1} cdot cdots cdot p_k^{r_k}$$
where the $p_i$s are distinct primes.



Given
$$n=pcdot d(n)=p(r+1)(r_1+1)cdots (r_k+1)$$
$$p^r cdots p_1^{r_1}cdot cdots cdot p_k^{r_k}=(r+1)(r_1+1)cdots(r_k+1)$$
If $k=0$, then $p^(r-1)=r+1$. Hence $p=2$ and $r=3$, or $p=3$ and $r=2$. I.e., $n=8$, or $n=9$.



But, when I am assuming $k>0$, I am finding no clue.



For this, I need help.



Thanks in advance!










share|cite|improve this question











$endgroup$












  • $begingroup$
    Why do you believe the list is sensible? For $n=84$ we get $7$. For $n=88$ we get $11$. For $n=156$ you get $13$. I stopped looking after that,
    $endgroup$
    – lulu
    Jan 20 at 14:33












  • $begingroup$
    then maybe you will get a general form, something like 4k, or 9k+ 1 numbers.... (i am giving an example)
    $endgroup$
    – user636268
    Jan 20 at 14:37










  • $begingroup$
    Seems that $9$ is the only odd such number (I have no proof yet), but there are plenty of even numbers satisfying the condition giving no obvious pattern except that $18$ seems to be the only even example which is not divisible by $4$.
    $endgroup$
    – Peter
    Jan 20 at 14:55












  • $begingroup$
    But this is a standard question from one of the most famous math institues of our country, Indian Statistical Institute.. Obviously there is something, which i am unable to make out
    $endgroup$
    – user636268
    Jan 20 at 14:57






  • 1




    $begingroup$
    I don't see where you have made a conjecture about numbers of the form $4k,8k$. Note: all numbers of the form $8p$ where $p$ is prime work (trivially).
    $endgroup$
    – lulu
    Jan 20 at 15:41
















5












$begingroup$



Let $d(n)$ denote the number of positive divisors of $n$. Find all $n$ such that $n/d(n) = p$, a prime.




I tried this, but only I could get two solutions.
I proceeded like this -



Suppose
$$n = p^r cdot p_1^{r_1} cdot cdots cdot p_k^{r_k}$$
where the $p_i$s are distinct primes.



Given
$$n=pcdot d(n)=p(r+1)(r_1+1)cdots (r_k+1)$$
$$p^r cdots p_1^{r_1}cdot cdots cdot p_k^{r_k}=(r+1)(r_1+1)cdots(r_k+1)$$
If $k=0$, then $p^(r-1)=r+1$. Hence $p=2$ and $r=3$, or $p=3$ and $r=2$. I.e., $n=8$, or $n=9$.



But, when I am assuming $k>0$, I am finding no clue.



For this, I need help.



Thanks in advance!










share|cite|improve this question











$endgroup$












  • $begingroup$
    Why do you believe the list is sensible? For $n=84$ we get $7$. For $n=88$ we get $11$. For $n=156$ you get $13$. I stopped looking after that,
    $endgroup$
    – lulu
    Jan 20 at 14:33












  • $begingroup$
    then maybe you will get a general form, something like 4k, or 9k+ 1 numbers.... (i am giving an example)
    $endgroup$
    – user636268
    Jan 20 at 14:37










  • $begingroup$
    Seems that $9$ is the only odd such number (I have no proof yet), but there are plenty of even numbers satisfying the condition giving no obvious pattern except that $18$ seems to be the only even example which is not divisible by $4$.
    $endgroup$
    – Peter
    Jan 20 at 14:55












  • $begingroup$
    But this is a standard question from one of the most famous math institues of our country, Indian Statistical Institute.. Obviously there is something, which i am unable to make out
    $endgroup$
    – user636268
    Jan 20 at 14:57






  • 1




    $begingroup$
    I don't see where you have made a conjecture about numbers of the form $4k,8k$. Note: all numbers of the form $8p$ where $p$ is prime work (trivially).
    $endgroup$
    – lulu
    Jan 20 at 15:41














5












5








5


1



$begingroup$



Let $d(n)$ denote the number of positive divisors of $n$. Find all $n$ such that $n/d(n) = p$, a prime.




I tried this, but only I could get two solutions.
I proceeded like this -



Suppose
$$n = p^r cdot p_1^{r_1} cdot cdots cdot p_k^{r_k}$$
where the $p_i$s are distinct primes.



Given
$$n=pcdot d(n)=p(r+1)(r_1+1)cdots (r_k+1)$$
$$p^r cdots p_1^{r_1}cdot cdots cdot p_k^{r_k}=(r+1)(r_1+1)cdots(r_k+1)$$
If $k=0$, then $p^(r-1)=r+1$. Hence $p=2$ and $r=3$, or $p=3$ and $r=2$. I.e., $n=8$, or $n=9$.



But, when I am assuming $k>0$, I am finding no clue.



For this, I need help.



Thanks in advance!










share|cite|improve this question











$endgroup$





Let $d(n)$ denote the number of positive divisors of $n$. Find all $n$ such that $n/d(n) = p$, a prime.




I tried this, but only I could get two solutions.
I proceeded like this -



Suppose
$$n = p^r cdot p_1^{r_1} cdot cdots cdot p_k^{r_k}$$
where the $p_i$s are distinct primes.



Given
$$n=pcdot d(n)=p(r+1)(r_1+1)cdots (r_k+1)$$
$$p^r cdots p_1^{r_1}cdot cdots cdot p_k^{r_k}=(r+1)(r_1+1)cdots(r_k+1)$$
If $k=0$, then $p^(r-1)=r+1$. Hence $p=2$ and $r=3$, or $p=3$ and $r=2$. I.e., $n=8$, or $n=9$.



But, when I am assuming $k>0$, I am finding no clue.



For this, I need help.



Thanks in advance!







number-theory prime-numbers divisibility prime-factorization






share|cite|improve this question















share|cite|improve this question













share|cite|improve this question




share|cite|improve this question








edited Jan 20 at 14:38









Blue

48.5k870154




48.5k870154










asked Jan 20 at 14:28







user636268



















  • $begingroup$
    Why do you believe the list is sensible? For $n=84$ we get $7$. For $n=88$ we get $11$. For $n=156$ you get $13$. I stopped looking after that,
    $endgroup$
    – lulu
    Jan 20 at 14:33












  • $begingroup$
    then maybe you will get a general form, something like 4k, or 9k+ 1 numbers.... (i am giving an example)
    $endgroup$
    – user636268
    Jan 20 at 14:37










  • $begingroup$
    Seems that $9$ is the only odd such number (I have no proof yet), but there are plenty of even numbers satisfying the condition giving no obvious pattern except that $18$ seems to be the only even example which is not divisible by $4$.
    $endgroup$
    – Peter
    Jan 20 at 14:55












  • $begingroup$
    But this is a standard question from one of the most famous math institues of our country, Indian Statistical Institute.. Obviously there is something, which i am unable to make out
    $endgroup$
    – user636268
    Jan 20 at 14:57






  • 1




    $begingroup$
    I don't see where you have made a conjecture about numbers of the form $4k,8k$. Note: all numbers of the form $8p$ where $p$ is prime work (trivially).
    $endgroup$
    – lulu
    Jan 20 at 15:41


















  • $begingroup$
    Why do you believe the list is sensible? For $n=84$ we get $7$. For $n=88$ we get $11$. For $n=156$ you get $13$. I stopped looking after that,
    $endgroup$
    – lulu
    Jan 20 at 14:33












  • $begingroup$
    then maybe you will get a general form, something like 4k, or 9k+ 1 numbers.... (i am giving an example)
    $endgroup$
    – user636268
    Jan 20 at 14:37










  • $begingroup$
    Seems that $9$ is the only odd such number (I have no proof yet), but there are plenty of even numbers satisfying the condition giving no obvious pattern except that $18$ seems to be the only even example which is not divisible by $4$.
    $endgroup$
    – Peter
    Jan 20 at 14:55












  • $begingroup$
    But this is a standard question from one of the most famous math institues of our country, Indian Statistical Institute.. Obviously there is something, which i am unable to make out
    $endgroup$
    – user636268
    Jan 20 at 14:57






  • 1




    $begingroup$
    I don't see where you have made a conjecture about numbers of the form $4k,8k$. Note: all numbers of the form $8p$ where $p$ is prime work (trivially).
    $endgroup$
    – lulu
    Jan 20 at 15:41
















$begingroup$
Why do you believe the list is sensible? For $n=84$ we get $7$. For $n=88$ we get $11$. For $n=156$ you get $13$. I stopped looking after that,
$endgroup$
– lulu
Jan 20 at 14:33






$begingroup$
Why do you believe the list is sensible? For $n=84$ we get $7$. For $n=88$ we get $11$. For $n=156$ you get $13$. I stopped looking after that,
$endgroup$
– lulu
Jan 20 at 14:33














$begingroup$
then maybe you will get a general form, something like 4k, or 9k+ 1 numbers.... (i am giving an example)
$endgroup$
– user636268
Jan 20 at 14:37




$begingroup$
then maybe you will get a general form, something like 4k, or 9k+ 1 numbers.... (i am giving an example)
$endgroup$
– user636268
Jan 20 at 14:37












$begingroup$
Seems that $9$ is the only odd such number (I have no proof yet), but there are plenty of even numbers satisfying the condition giving no obvious pattern except that $18$ seems to be the only even example which is not divisible by $4$.
$endgroup$
– Peter
Jan 20 at 14:55






$begingroup$
Seems that $9$ is the only odd such number (I have no proof yet), but there are plenty of even numbers satisfying the condition giving no obvious pattern except that $18$ seems to be the only even example which is not divisible by $4$.
$endgroup$
– Peter
Jan 20 at 14:55














$begingroup$
But this is a standard question from one of the most famous math institues of our country, Indian Statistical Institute.. Obviously there is something, which i am unable to make out
$endgroup$
– user636268
Jan 20 at 14:57




$begingroup$
But this is a standard question from one of the most famous math institues of our country, Indian Statistical Institute.. Obviously there is something, which i am unable to make out
$endgroup$
– user636268
Jan 20 at 14:57




1




1




$begingroup$
I don't see where you have made a conjecture about numbers of the form $4k,8k$. Note: all numbers of the form $8p$ where $p$ is prime work (trivially).
$endgroup$
– lulu
Jan 20 at 15:41




$begingroup$
I don't see where you have made a conjecture about numbers of the form $4k,8k$. Note: all numbers of the form $8p$ where $p$ is prime work (trivially).
$endgroup$
– lulu
Jan 20 at 15:41










1 Answer
1






active

oldest

votes


















2












$begingroup$

Observation 1. It is easy to prove (e.g. by induction) that:
$$p^n geq n+1, forall pgeq2, forall ngeq 1$$
Thus
$$p^rcdot color{blue}{p_1^{r_1}cdot ... cdot p_k^{r_k}}=
p(r+1)cdot color{blue}{(r_1+1)...(r_k+1)}geq
p^r cdot color{blue}{(r_1+1)...(r_k+1)}$$

or
$$p(r+1)geq p^r iff r+1geq p^{r-1}$$
These are the only $(r,p)$ combinations possible $(1,2), (2,2), (3,2)$ then $(1,3),(2,3)$ and finally $(1,p), forall p>3, p$ - prime.





Observation 2. Let's check the following case $(1,p), forall p>3, p$ - prime (i.e. $r=1$).



$$pcdot color{blue}{p_1^{r_1}cdot ... cdot p_k^{r_k}}=
pcdot 2cdot color{blue}{(r_1+1)...(r_k+1)} iff \
p_1^{r_1}cdot ... cdot p_k^{r_k}=
2cdot (r_1+1)...(r_k+1) iff ...$$

or one of the $p_i=2$, for simplicity let's say $p_1=2$.
$$... iff 2^{r_1-1}cdot color{blue}{p_2^{r_2} ... cdot p_k^{r_k}}=
(r_1+1)color{blue}{(r_2+1)...(r_k+1)}geq
2^{r_1-1}cdot color{blue}{(r_2+1)...(r_k+1)}$$

or again
$$(r_1+1)geq 2^{r_1-1}$$
or $r_1 in {1,2,3}$.



This reduces the problem to the following cases $(1,2), (2,2), (3,2)$ then $(1,3),(2,3)$.






share|cite|improve this answer











$endgroup$













  • $begingroup$
    Wrong solution. Check 8,18
    $endgroup$
    – user636268
    Jan 20 at 16:29










  • $begingroup$
    Which solution? This is an observation which narrows down the search space.
    $endgroup$
    – rtybase
    Jan 20 at 16:30






  • 1




    $begingroup$
    Sorry i didnt read that
    $endgroup$
    – user636268
    Jan 20 at 16:31











Your Answer





StackExchange.ifUsing("editor", function () {
return StackExchange.using("mathjaxEditing", function () {
StackExchange.MarkdownEditor.creationCallbacks.add(function (editor, postfix) {
StackExchange.mathjaxEditing.prepareWmdForMathJax(editor, postfix, [["$", "$"], ["\\(","\\)"]]);
});
});
}, "mathjax-editing");

StackExchange.ready(function() {
var channelOptions = {
tags: "".split(" "),
id: "69"
};
initTagRenderer("".split(" "), "".split(" "), channelOptions);

StackExchange.using("externalEditor", function() {
// Have to fire editor after snippets, if snippets enabled
if (StackExchange.settings.snippets.snippetsEnabled) {
StackExchange.using("snippets", function() {
createEditor();
});
}
else {
createEditor();
}
});

function createEditor() {
StackExchange.prepareEditor({
heartbeatType: 'answer',
autoActivateHeartbeat: false,
convertImagesToLinks: true,
noModals: true,
showLowRepImageUploadWarning: true,
reputationToPostImages: 10,
bindNavPrevention: true,
postfix: "",
imageUploader: {
brandingHtml: "Powered by u003ca class="icon-imgur-white" href="https://imgur.com/"u003eu003c/au003e",
contentPolicyHtml: "User contributions licensed under u003ca href="https://creativecommons.org/licenses/by-sa/3.0/"u003ecc by-sa 3.0 with attribution requiredu003c/au003e u003ca href="https://stackoverflow.com/legal/content-policy"u003e(content policy)u003c/au003e",
allowUrls: true
},
noCode: true, onDemand: true,
discardSelector: ".discard-answer"
,immediatelyShowMarkdownHelp:true
});


}
});














draft saved

draft discarded


















StackExchange.ready(
function () {
StackExchange.openid.initPostLogin('.new-post-login', 'https%3a%2f%2fmath.stackexchange.com%2fquestions%2f3080643%2ffind-all-n-such-that-n-dn-p-a-prime-where-dn-is-the-number-of-posi%23new-answer', 'question_page');
}
);

Post as a guest















Required, but never shown
























1 Answer
1






active

oldest

votes








1 Answer
1






active

oldest

votes









active

oldest

votes






active

oldest

votes









2












$begingroup$

Observation 1. It is easy to prove (e.g. by induction) that:
$$p^n geq n+1, forall pgeq2, forall ngeq 1$$
Thus
$$p^rcdot color{blue}{p_1^{r_1}cdot ... cdot p_k^{r_k}}=
p(r+1)cdot color{blue}{(r_1+1)...(r_k+1)}geq
p^r cdot color{blue}{(r_1+1)...(r_k+1)}$$

or
$$p(r+1)geq p^r iff r+1geq p^{r-1}$$
These are the only $(r,p)$ combinations possible $(1,2), (2,2), (3,2)$ then $(1,3),(2,3)$ and finally $(1,p), forall p>3, p$ - prime.





Observation 2. Let's check the following case $(1,p), forall p>3, p$ - prime (i.e. $r=1$).



$$pcdot color{blue}{p_1^{r_1}cdot ... cdot p_k^{r_k}}=
pcdot 2cdot color{blue}{(r_1+1)...(r_k+1)} iff \
p_1^{r_1}cdot ... cdot p_k^{r_k}=
2cdot (r_1+1)...(r_k+1) iff ...$$

or one of the $p_i=2$, for simplicity let's say $p_1=2$.
$$... iff 2^{r_1-1}cdot color{blue}{p_2^{r_2} ... cdot p_k^{r_k}}=
(r_1+1)color{blue}{(r_2+1)...(r_k+1)}geq
2^{r_1-1}cdot color{blue}{(r_2+1)...(r_k+1)}$$

or again
$$(r_1+1)geq 2^{r_1-1}$$
or $r_1 in {1,2,3}$.



This reduces the problem to the following cases $(1,2), (2,2), (3,2)$ then $(1,3),(2,3)$.






share|cite|improve this answer











$endgroup$













  • $begingroup$
    Wrong solution. Check 8,18
    $endgroup$
    – user636268
    Jan 20 at 16:29










  • $begingroup$
    Which solution? This is an observation which narrows down the search space.
    $endgroup$
    – rtybase
    Jan 20 at 16:30






  • 1




    $begingroup$
    Sorry i didnt read that
    $endgroup$
    – user636268
    Jan 20 at 16:31
















2












$begingroup$

Observation 1. It is easy to prove (e.g. by induction) that:
$$p^n geq n+1, forall pgeq2, forall ngeq 1$$
Thus
$$p^rcdot color{blue}{p_1^{r_1}cdot ... cdot p_k^{r_k}}=
p(r+1)cdot color{blue}{(r_1+1)...(r_k+1)}geq
p^r cdot color{blue}{(r_1+1)...(r_k+1)}$$

or
$$p(r+1)geq p^r iff r+1geq p^{r-1}$$
These are the only $(r,p)$ combinations possible $(1,2), (2,2), (3,2)$ then $(1,3),(2,3)$ and finally $(1,p), forall p>3, p$ - prime.





Observation 2. Let's check the following case $(1,p), forall p>3, p$ - prime (i.e. $r=1$).



$$pcdot color{blue}{p_1^{r_1}cdot ... cdot p_k^{r_k}}=
pcdot 2cdot color{blue}{(r_1+1)...(r_k+1)} iff \
p_1^{r_1}cdot ... cdot p_k^{r_k}=
2cdot (r_1+1)...(r_k+1) iff ...$$

or one of the $p_i=2$, for simplicity let's say $p_1=2$.
$$... iff 2^{r_1-1}cdot color{blue}{p_2^{r_2} ... cdot p_k^{r_k}}=
(r_1+1)color{blue}{(r_2+1)...(r_k+1)}geq
2^{r_1-1}cdot color{blue}{(r_2+1)...(r_k+1)}$$

or again
$$(r_1+1)geq 2^{r_1-1}$$
or $r_1 in {1,2,3}$.



This reduces the problem to the following cases $(1,2), (2,2), (3,2)$ then $(1,3),(2,3)$.






share|cite|improve this answer











$endgroup$













  • $begingroup$
    Wrong solution. Check 8,18
    $endgroup$
    – user636268
    Jan 20 at 16:29










  • $begingroup$
    Which solution? This is an observation which narrows down the search space.
    $endgroup$
    – rtybase
    Jan 20 at 16:30






  • 1




    $begingroup$
    Sorry i didnt read that
    $endgroup$
    – user636268
    Jan 20 at 16:31














2












2








2





$begingroup$

Observation 1. It is easy to prove (e.g. by induction) that:
$$p^n geq n+1, forall pgeq2, forall ngeq 1$$
Thus
$$p^rcdot color{blue}{p_1^{r_1}cdot ... cdot p_k^{r_k}}=
p(r+1)cdot color{blue}{(r_1+1)...(r_k+1)}geq
p^r cdot color{blue}{(r_1+1)...(r_k+1)}$$

or
$$p(r+1)geq p^r iff r+1geq p^{r-1}$$
These are the only $(r,p)$ combinations possible $(1,2), (2,2), (3,2)$ then $(1,3),(2,3)$ and finally $(1,p), forall p>3, p$ - prime.





Observation 2. Let's check the following case $(1,p), forall p>3, p$ - prime (i.e. $r=1$).



$$pcdot color{blue}{p_1^{r_1}cdot ... cdot p_k^{r_k}}=
pcdot 2cdot color{blue}{(r_1+1)...(r_k+1)} iff \
p_1^{r_1}cdot ... cdot p_k^{r_k}=
2cdot (r_1+1)...(r_k+1) iff ...$$

or one of the $p_i=2$, for simplicity let's say $p_1=2$.
$$... iff 2^{r_1-1}cdot color{blue}{p_2^{r_2} ... cdot p_k^{r_k}}=
(r_1+1)color{blue}{(r_2+1)...(r_k+1)}geq
2^{r_1-1}cdot color{blue}{(r_2+1)...(r_k+1)}$$

or again
$$(r_1+1)geq 2^{r_1-1}$$
or $r_1 in {1,2,3}$.



This reduces the problem to the following cases $(1,2), (2,2), (3,2)$ then $(1,3),(2,3)$.






share|cite|improve this answer











$endgroup$



Observation 1. It is easy to prove (e.g. by induction) that:
$$p^n geq n+1, forall pgeq2, forall ngeq 1$$
Thus
$$p^rcdot color{blue}{p_1^{r_1}cdot ... cdot p_k^{r_k}}=
p(r+1)cdot color{blue}{(r_1+1)...(r_k+1)}geq
p^r cdot color{blue}{(r_1+1)...(r_k+1)}$$

or
$$p(r+1)geq p^r iff r+1geq p^{r-1}$$
These are the only $(r,p)$ combinations possible $(1,2), (2,2), (3,2)$ then $(1,3),(2,3)$ and finally $(1,p), forall p>3, p$ - prime.





Observation 2. Let's check the following case $(1,p), forall p>3, p$ - prime (i.e. $r=1$).



$$pcdot color{blue}{p_1^{r_1}cdot ... cdot p_k^{r_k}}=
pcdot 2cdot color{blue}{(r_1+1)...(r_k+1)} iff \
p_1^{r_1}cdot ... cdot p_k^{r_k}=
2cdot (r_1+1)...(r_k+1) iff ...$$

or one of the $p_i=2$, for simplicity let's say $p_1=2$.
$$... iff 2^{r_1-1}cdot color{blue}{p_2^{r_2} ... cdot p_k^{r_k}}=
(r_1+1)color{blue}{(r_2+1)...(r_k+1)}geq
2^{r_1-1}cdot color{blue}{(r_2+1)...(r_k+1)}$$

or again
$$(r_1+1)geq 2^{r_1-1}$$
or $r_1 in {1,2,3}$.



This reduces the problem to the following cases $(1,2), (2,2), (3,2)$ then $(1,3),(2,3)$.







share|cite|improve this answer














share|cite|improve this answer



share|cite|improve this answer








edited Jan 20 at 17:18

























answered Jan 20 at 16:28









rtybasertybase

11k21533




11k21533












  • $begingroup$
    Wrong solution. Check 8,18
    $endgroup$
    – user636268
    Jan 20 at 16:29










  • $begingroup$
    Which solution? This is an observation which narrows down the search space.
    $endgroup$
    – rtybase
    Jan 20 at 16:30






  • 1




    $begingroup$
    Sorry i didnt read that
    $endgroup$
    – user636268
    Jan 20 at 16:31


















  • $begingroup$
    Wrong solution. Check 8,18
    $endgroup$
    – user636268
    Jan 20 at 16:29










  • $begingroup$
    Which solution? This is an observation which narrows down the search space.
    $endgroup$
    – rtybase
    Jan 20 at 16:30






  • 1




    $begingroup$
    Sorry i didnt read that
    $endgroup$
    – user636268
    Jan 20 at 16:31
















$begingroup$
Wrong solution. Check 8,18
$endgroup$
– user636268
Jan 20 at 16:29




$begingroup$
Wrong solution. Check 8,18
$endgroup$
– user636268
Jan 20 at 16:29












$begingroup$
Which solution? This is an observation which narrows down the search space.
$endgroup$
– rtybase
Jan 20 at 16:30




$begingroup$
Which solution? This is an observation which narrows down the search space.
$endgroup$
– rtybase
Jan 20 at 16:30




1




1




$begingroup$
Sorry i didnt read that
$endgroup$
– user636268
Jan 20 at 16:31




$begingroup$
Sorry i didnt read that
$endgroup$
– user636268
Jan 20 at 16:31


















draft saved

draft discarded




















































Thanks for contributing an answer to Mathematics Stack Exchange!


  • Please be sure to answer the question. Provide details and share your research!

But avoid



  • Asking for help, clarification, or responding to other answers.

  • Making statements based on opinion; back them up with references or personal experience.


Use MathJax to format equations. MathJax reference.


To learn more, see our tips on writing great answers.




draft saved


draft discarded














StackExchange.ready(
function () {
StackExchange.openid.initPostLogin('.new-post-login', 'https%3a%2f%2fmath.stackexchange.com%2fquestions%2f3080643%2ffind-all-n-such-that-n-dn-p-a-prime-where-dn-is-the-number-of-posi%23new-answer', 'question_page');
}
);

Post as a guest















Required, but never shown





















































Required, but never shown














Required, but never shown












Required, but never shown







Required, but never shown

































Required, but never shown














Required, but never shown












Required, but never shown







Required, but never shown







Popular posts from this blog

Mario Kart Wii

What does “Dominus providebit” mean?

Antonio Litta Visconti Arese